2 Exercise Two – Jurisdiction

I. THE LAW OF JURISDICTION

A. Types of Jurisdiction

 

Before suit can be brought in a given forum, the court must have “jurisdiction,” the power to speak (diction) the law (juris). Jurisdiction is divided into three components: 1) subject matter jurisdiction, 2) personal jurisdiction, and 3) notice and opportunity to be heard. The court needs all three types of jurisdiction to proceed to adjudicate the lawsuit. In addition, venue must be properly laid: plaintiff must bring the suit in a proper district.

Subject matter jurisdiction is a question of authority of the court over the nature of the litigation, i.e., the subject matter presented. The rules of subject matter jurisdiction ask whether the court has been given power to decide a certain type of legal controversy. In order to determine whether a federal court possesses subject matter jurisdiction, a lawyer will look to the Constitution of the United States and federal statutes; for a state court, the lawyer will look to the state constitution and state statutes.

Many state courts are given broad power to hear all types of cases. They are courts of general subject matter jurisdiction. Other courts are limited in the types of cases they may hear, and are courts of limited subject matter jurisdiction. The common limits are the type of claim or the amount claimed. For example, a state might have a court of general jurisdiction, plus a specific court to deal with probate law, another to deal with tax cases, and a third to deal with workers’ compensation cases. In such a system, a suit on a contract brought in probate court would be dismissed for lack of subject matter jurisdiction. Another state might establish a small claims court to hear cases not exceeding an amount in controversy of $5,000. A case seeking in excess of $5,000, brought in that court, would be dismissed for lack of subject matter jurisdiction.

All federal courts–district courts, courts of appeals, and the Supreme Court of the United States–are courts of limited subject matter jurisdiction. The limits on federal jurisdiction are discussed in I.B.2, infra. Because federal courts have limited subject matter jurisdiction, a party seeking to enter federal court must specifically plead that jurisdiction. See II.B.1, infra.

Because subject matter jurisdiction is conferred by constitution and statute, it cannot be conferred on a court by the parties. The parties can neither consent to nor waive subject matter jurisdiction. The federal court rule that subject matter jurisdiction may be challenged at any stage of the litigation [see Fed. R. Civ. P. 12(h)(3)] necessarily follows from this principle.

Personal jurisdiction refers to the court’s power over the parties, or over the parties’ property, in the case. Some writers prefer to call this type of jurisdiction basis jurisdiction, but the more common usage is personal jurisdiction, which as we use the term, includes power over both persons and things. Since a plaintiff who commences an action consents to the personal jurisdiction of the court, the question is power over defendants.

When the court possesses power over the person of the defendant, the court has in personam jurisdiction. The traditional method for a court to obtain power over the person of a defendant has always been service on that person within the boundaries of the state. The human or corporate defendant had to be “present” within the state for the court to be able to exercise power. As twentieth century advances in transportation and commerce made commercial activity far from home common, courts began to shift emphasis away from physical presence in the state to a consideration of whether the state was a convenient geographic location for the lawsuit. States enacted long-arm statutes to reach out beyond state boundaries to bring defendants into the state from afar. Even today, however, convenience has not become the sole consideration for jurisdiction; a state is required to refrain from exercising jurisdiction over a defendant who has insufficient “minimum contacts” with the forum state, even though the forum would be a convenient location for the lawsuit. See I.C.1(d), infra.

Personal jurisdiction is broader than in personam jurisdiction, since the court’s power may be based not directly on the person of the defendant, but on the defendant through property owned. When the purpose of the litigation is to determine rights in a piece of property, the res, located in the state, the action is an in rem proceeding. When the claim in the litigation is unrelated to the property, and the plaintiff seizes the res for the sole purpose of obtaining jurisdiction over the defendant within the state, the jurisdiction is quasi in rem. The court’s power over the person of the defendant will extend only to the limits of the value of the defendant’s interest in the seized property. See I.C.2, infra.

Since personal jurisdiction means power over the person or the property of the defendant, defendant can confer it on the court by consent or waiver. A personal jurisdiction defense not raised by defendant at the first opportunity is waived [see Fed. R. Civ. P. 12(h)(1)]. This possibility of waiver is explored in detail in Exercise Five: Motions to Dismiss and Waiver Under Federal Rule 12.

Notice and opportunity to be heard form the third part of the jurisdictional triangle. The defendant must receive notice of the pending action and a meaningful opportunity to be heard in defense before defendant’s property is taken. Due process requires no less. Notice is accomplished by service of a summons on the defendant, whether by personal service in hand, substituted service on another for the defendant, or constructive service of publication of the summons in a newspaper. Of these methods, the problematical one is service by publication, since that method may not be reasonably calculated to achieve actual notice to defendant. See Mullane v. Central Hanover Bank & Trust Co., 339 U.S. 306, 315, 70 S. Ct. 652, 657, 94 L.Ed. 865, 874 (1950) (“The means employed must be … reasonably certain to inform those affected.”).

Note that personal service on a defendant out of state cannot raise an objection to notice: defendant certainly has received proper notice, even though the defendant might have a valid objection to personal jurisdiction. Finally, since notice may be received even through improper procedures, the defendant may consent to notice.

Opportunity to be heard problems arise in two situations. First, and rarely, defendant is given an inadequate time to respond to the complaint. Second, more commonly, property of the defendant is seized prior to judgment; defendant must receive a meaningful chance to defend the case before, or at least shortly after, the property is taken.[i]

Venue is not a jurisdictional requirement, yet it is closely associated with jurisdiction and the two are sometimes confused. The difference is that jurisdiction deals with the power of the court, while venue allocates judicial business among various courts that have jurisdictional power. Should one federal district court have subject matter jurisdiction over a case, all 94 federal district courts will have subject matter jurisdiction. Every one of them will be able to give proper notice. Perhaps even all of them will have personal jurisdiction over the defendant. Yet the federal venue statutes (see section I.E.2) will require that the case be brought only in a small number, or even only one, of those 94 districts. The rules of venue are designed to ensure that trials are conducted in a convenient place.

A case laying venue in the wrong federal district may be dismissed [Fed. R. Civ. P. 12(b)(3)] or may be transferred to a district “in which it could have been brought” [28 U.S.C. § 1406]. A defendant may also consent or waive objection to venue [Fed. R. Civ. P. 12(h)(1); see Exercise Five: Motions to Dismiss and Waiver Under Federal Rule 12].

B. Subject Matter Jurisdiction

1. State Courts

Even though state court systems do have courts of limited subject matter jurisdiction, every state has a state court of general subject matter jurisdiction that is able to hear all types of cases. That court may be called the superior court, the district court, the supreme court, the circuit court, or some other name. You may assume that all state courts mentioned in this exercise are courts of general jurisdiction.

2. Federal Courts

Since subject matter jurisdiction is dependent on the constitution and statutes that empower the court, we look first to the Judicial Article of the United States Constitution to determine what types of cases it permits federal courts to hear:

The judicial power shall extend to all Cases, in Law and Equity, arising under this Constitution, the Laws of the United States, and Treaties made, or which shall be made, under their Authority;—to all Cases affecting Ambassadors, other public Ministers and Consuls;—to all Cases of admiralty and maritime Jurisdiction;—to controversies to which the United States shall be a Party;—to Controversies between two or more States;—between a State and Citizens of another State;—between Citizens of different States;—between Citizens of the same State claiming Lands under Grants of different States, and between a State, or the Citizens thereof, and foreign States, Citizens or Subjects.

U.S.Const. art. III, § 2. The two most commonly-used types of federal court subject matter jurisdiction are federal questions [“all Cases, in Law and Equity, arising under this Constitution, the laws of the United States, and Treaties made”] and diversity of citizenship [“between Citizens of different States”].

Even though the Constitution provides the federal judicial power “shall extend” over these two areas, Congress must pass legislation to vest such authority in the federal courts. While the matter is not free from doubt, the Supreme Court has often said that the judicial power of the United States—within the outer limits of the Constitution—is dependent on Congress, which may invest the inferior federal courts “with jurisdiction either limited, concurrent, or exclusive, and [withhold] jurisdiction from them in the exact degrees and character which to Congress may seem proper for the public good.” Sheldon v. Sill, 49 U.S. (8 How.) 441, 12 L.Ed. 1147 (1850). Indeed, Congress did not grant the federal courts authority to hear federal question cases until 1875, and proposals to eliminate diversity jurisdiction are even today introduced.

a. Federal Question Jurisdiction

The statute implementing the Constitutional grant of power over federal questions reads as follows:

The district courts shall have original jurisdiction of all civil actions arising under the Constitution, laws, or treaties of the United States.

28 U.S.C. § 1331.

A case arising under the Constitution, laws, or treaties of the United States presents a federal question. Even though the key words of the statute–“arising under”–are identical to the Constitution, the courts interpret the statute more narrowly than the Constitution. For example, the famous rule that a federal question must arise in the well‑pleaded complaint and cannot arise in defense [Louisville & Nashville R.R. v. Mottley, 211 U.S. 149, 29 S. Ct. 42, 53 L.Ed. 126 (1908), reproduced in Exercise One] is recognized as an interpretation of § 1331 but not of the Constitution.

When a federal statute creates the claim, courts have little difficulty recognizing a federal question. Even here, however, what appears at first glance to be a claim founded on federal law may not be. Suppose, for example, that an author assigns a copyright issued under federal copyright law to an entrepreneur in exchange for an agreement to share royalties on marketing of the composition. Some time later, the author sues for failure to pay royalties and seeks an accounting. Is this a federal question? No, said Judge Henry Friendly in T.B. Harms Co. v. Eliscu, 339 F.2d 823, 828 (2d Cir.1964): the complaint presents a state law claim of breach of contract.

The most difficult cases have been those in which a federal law issue arises in a claim created by state law. Despite the best efforts of some of the best judges of our national history—Chief Justice John Marshall, Justice Oliver Wendell Holmes, and Justice Benjamin Cardozo among them—the phrase “arising under” has remained elusive in these cases. Perhaps the most famous attempt at definition was offered many years ago by Justice Cardozo:

If we follow the ascent far enough, countless claims of right can be discovered to have their source or their operative limits in the provisions of a federal statute or in the Constitution itself with its circumambient restrictions upon legislative power. To set bounds to the pursuit, the courts have formulated the distinction between controversies that are basic and those that are collateral, between disputes that are necessary and those that are merely possible. We shall be lost in a maze if we put that compass by.

Gully v. First Nat. Bank in Meridian, 299 U.S. 109, 118, 57 S. Ct. 96, 100, 81 L.Ed. 70, 75 (1936). Most recently, the Court attempted a new definition in Grable & Sons Prods., Inc. v. Darue Eng. & Mfg., 545 U.S. 308, 125 S. Ct. 2363, 162 L.Ed.2d 257 (2005).

At the ends of the continuum, cases can easily be classified as arising under federal law or arising under state law. Near the middle, the question of whether a case presents a federal question can be exceedingly difficult.[ii]

b. Diversity Jurisdiction

Diversity cases involve citizens of different states. The statute implementing the Constitutional grant of power over diversity of citizenship cases reads as follows:

(a) The district courts shall have original jurisdiction of all civil actions where the matter in controversy exceeds the sum or value of $75,000, exclusive of interest and costs, and is between—

(1) citizens of different States;

(2) citizens of a State and citizens or subjects of a foreign state;

(3) citizens of different States and in which citizens or subjects of a foreign state are additional parties; and

(4) a foreign state, defined in section 1603(a) of this title, as plaintiff and citizens of a State or of different States.

For the purposes of this section, section 1335, and section 1441, an alien admitted to the United States for permanent residence shall be deemed a citizen of the State in which such alien is domiciled.

(b) Except when express provision therefor is otherwise made in a statute of the United States, where the plaintiff who files the case originally in the Federal courts is finally adjudged to be entitled to recover less than the sum or value of $75,000, computed without regard to any setoff or counterclaim to which the defendant may be adjudged to be entitled, and exclusive of interest and costs, the district court may deny costs to the plaintiff and, in addition, may impose costs on the plaintiff.

(c) For the purposes of this section and section 1441 of this title—

(1) a corporation shall be deemed a citizen of any State by which it has been incorporated and of the State where it has its principal place of business,[iii] except that in any direct action against the insurer of a policy or contract of liability insurance, whether incorporated or unincorporated, to which action the insured is not joined as a party‑defendant, such insurer shall be deemed a citizen of the State of which the insured is a citizen, as well as of any State by which the insurer has been incorporated and of the State where it has its principal place of business; and
(2) the legal representative of the estate of a decedent shall be deemed to be a citizen only of the same State as the decedent, and the legal representative of an infant or incompetent shall be deemed to be a citizen only of the same State as the infant or incompetent.

(d) [provisions relating to class actions omitted]

(e) The word “States”, as used in this section, includes the Territories, the District of Columbia, and the Commonwealth of Puerto Rico.

28 U.S.C. § 1332.

__________

This statute clearly requires that for a federal court to have diversity jurisdiction, the case must satisfy two requirements: 1) diversity of citizenship of the parties and 2) an adequate jurisdictional amount.

1. Determining Citizenship of the Parties. While 28 U.S.C. § 1332 states at least a partial test for determining the citizenship of a corporation, it offers no guidance for determining the citizenship of a natural person–or other entities such as partnerships or unions. The following two cases exemplify the law that federal courts have created to determine the citizenship of a natural person. Please read them carefully, as you will be called on to apply the principles announced in them, and you will also be asked questions specifically about the opinions, when you do the computer exercise.

BAKER v. KECK
United States District Court, Eastern District of Illinois, 1936.
13 F. Supp. 486.

[This opinion is reproduced in Exercise One, part III. Please reread Baker as part of your preparation for this exercise. Remember that even though the opinion is old, it still states good law today and you should treat it as a recent decision.]

SCOGGINS v. POLLOCK
United States Court of Appeals, Eleventh Circuit, 1984.
727 F.2d 1025.

Albert J. Henderson, Circuit Judge:

The issue presented by this case is whether the appellant, Kay Scoggins, was domiciled in South Carolina or Georgia at the time she filed this medical malpractice suit in the United States District Court for the Southern District of Georgia. Diversity of citizenship is alleged as the basis of federal jurisdiction pursuant to the provisions of 28 U.S.C. § 1332. All of the appellees‑defendants are residents of Georgia. The district court concluded that Mrs. Scoggins was also a citizen of Georgia and dismissed the case for lack of subject matter jurisdiction. Finding that the district court was not clearly erroneous, we affirm.

Mrs. Scoggins and her husband lived in Washington, Georgia. He was a high school principal and Mrs. Scoggins worked as a media specialist in a grade school. As a result of Mr. Scoggins’ sudden death in October, 1979, Mrs. Scoggins filed suit in October, 1981 against the doctors, clinic and hospital that treated him.

Mrs. Scoggins remained in Washington, Georgia for over a year after her husband’s death. Rev. Robert Murphy, who counseled with her, stated that he advised her not to do anything for at least a year until she overcame her grief. Rev. Murphy Deposition at 13. Still, Mrs. Scoggins contended that she decided soon after her husband’s death to leave Washington and start a new life somewhere else. Mrs. Scoggins Deposition at 131.

In January or February 1981 Mrs. Scoggins applied for admission to a one year Master in Librarianship program at the University of South Carolina. After her acceptance in mid‑April, 1981, she notified her employer, Dr. Fred Dorminy, of her intent to resign her job in the Wilkes County school system. She rented an apartment in West Columbia, South Carolina and began her course of study in August, 1981. Later she accepted a job as a graduate assistant, a position open only to students. She neither sold nor rented her house in Washington, Georgia. She and her two children stayed there occasionally when they were in Washington. She claimed that she was holding on to the house until she graduated and found a permanent job and then would use the proceeds of the sale to purchase a new home. Mrs. Scoggins Deposition at 132–33.

The district court correctly noted that a change of domicile requires “[a] concurrent showing of (1) physical presence at the new location with (2) an intention to remain there indefinitely….” Opinion at 4. Mas v. Perry, 489 F.2d 1396, 1399 (5th Cir.), cert. denied, 419 U.S. 842, 95 S. Ct. 74, 42 L.Ed.2d 70 (1974); Stine v. Moore, 213 F.2d 446 (5th Cir.1954). The plaintiff bears the burden of proving her domicile by a preponderance of the evidence. Vacca v. Meetze, 499 F. Supp. 1089 (S.D.Ga.1980).

It is undisputed that Mrs. Scoggins was physically present in South Carolina when she filed this suit. She had rented an apartment, registered to vote, registered her car and obtained a South Carolina driver’s license. After a summer vacation she apparently was in South Carolina full time once classes began. The second element of the test, her intent to remain in South Carolina indefinitely, however, presents a greater problem. The district court found that she initially went to South Carolina to undertake graduate studies and had not positively decided upon her residence after graduation. Citing 13 Wright, Miller & Cooper, Federal Practice and Procedure § 3613 (1975), the district court stated that out‑of‑state students are usually regarded only as temporary residents and “[i]t is therefore usually presumed that they retain their domicile at their former place of abode.” Opinion at 10. Because Mrs. Scoggins lacked the requisite intent to remain in South Carolina and was still a Georgia domiciliary, the district court then dismissed the suit for lack of jurisdiction.

The district court’s finding of domicile will not be disturbed unless clearly erroneous. Combee v. Shell Oil Co., 615 F.2d 698 (5th Cir.1980). We conclude that, although there is some conflicting evidence, we are not “left with the ‘definite and firm conviction that a mistake has been committed.’ ” Inwood Laboratories v. Ives Laboratories, 456 U.S. 844, 855, 102 S. Ct. 2182, 2189, 72 L.Ed.2d 606, 616 (1982) (quoting United States v. United States Gypsum Co., 333 U.S. 364, 395, 68 S. Ct. 525, 542, 92 L.Ed. 746, 766 (1948)). There is sufficient evidence in the record to support the district court’s finding.

Mrs. Scoggins clearly intended to leave Washington, Georgia, but her plans after that were more nebulous. Instead of consistently exhibiting an intent to remain in South Carolina, there were many indications that she considered moving to Florida or even returning to Georgia. Rev. Murphy stated that he discussed cities like Atlanta with Mrs. Scoggins and that “[s]he did name to me on more than one occasion that she was having thoughts of perhaps teaching in Florida.” Rev. Murphy Deposition at 15. Further,

she never indicated to me that she had made any plans to settle in South Carolina, nor did she say she didn’t. The fact is, she didn’t exclude Georgia really in her conversations to me.

Id. at 24. Dr. Dorminy, the county school superintendent in Wilkes County, testified by deposition that Mrs. Scoggins “indicated that when she finished her work at the University of South Carolina, that Florida was a possibility.” Dorminy Deposition at 17. Dr. Dorminy additionally remarked that she told him she was unsure where she would go after graduation and she did not say anything to him suggesting that she considered South Carolina as her permanent home. Id. at 13, 17.

Mrs. Scoggins herself testified that her plans at the time were unsettled. “My intentions were to leave Georgia. I really didn’t know where I was going, but I intended to leave Washington. I did not intend to live there any longer. I had options of where to go.” Mrs. Scoggins Deposition at 160. Initially, it appears that she went to South Carolina solely to pursue her graduate studies. The University of South Carolina offered one of the few accredited programs in which she was interested. When asked when she decided to move to South Carolina, she replied “[a]fter I received my acceptance from the University of South Carolina.” Id. at 134. Also,

Q. The only reason you went was to go to school then—you didn’t go to Columbia for any other reason, except to go to the University of South Carolina, is that correct?

A. That’s where I was accepted, so that’s why I am in Columbia, South Carolina.

Id. at 158.

The former Fifth Circuit Court of Appeals considered this precise question in Mas v. Perry, 489 F.2d 1396 (5th Cir.), cert. denied, 419 U.S. 842, 95 S. Ct. 74, 42 L.Ed.2d 70 (1974). [Note: the Eleventh Circuit Court of Appeals adopts as precedents the decisions of the Fifth Circuit Court of Appeals issued prior to the division of that court into the Fifth and Eleventh Circuits in 1981.] Mr. and Mrs. Mas were graduate students at Louisiana State University and worked as graduate assistants. Mrs. Mas previously lived in Mississippi. The couple moved to Illinois but planned to return to Louisiana for Mr. Mas to complete his doctorate degree. They each sued their former landlord who was a Louisiana resident. He challenged diversity jurisdiction over Mrs. Mas’ claim, alleging that Mrs. Mas also was domiciled in Louisiana. The court rejected this argument, stating:

Mrs. Mas’ Mississippi domicile was disturbed neither by her year in Louisiana prior to her marriage nor as a result of the time she and her husband spent at LSU after their marriage, since for both periods she was a graduate assistant at LSU. Though she testified that after her marriage she had no intention of returning to her parents’ home in Mississippi, Mrs. Mas did not effect a change of domicile since she and Mr. Mas were in Louisiana only as students and lacked the requisite intention to remain there.

Id. at 1400.

Mas is directly on point. Although Mrs. Scoggins may now intend to remain in South Carolina, we must look to the facts as of the date she filed this suit. She initially moved to South Carolina as a student. Even if she did not intend to return to Georgia, she was undecided about her future plans. Her domicile before she moved to South Carolina continued until she obtained a new one. Georgia remained Mrs. Scoggins’ domicile for diversity purposes. We also note that the Court of Appeals for the Eighth Circuit, also in a medical malpractice case, dismissed the diversity suit of a student in Ohio against a Missouri doctor, holding that the student retained his Missouri domicile because he lacked the intent to remain in Ohio. Holmes v. Sopuch, 639 F.2d 431 (8th Cir.1981). The district court’s finding that Mrs. Scoggins was a Georgia domiciliary is supported by the record and is not clearly erroneous.

Mrs. Scoggins also asserts that the district court abused its discretion by denying her motion to amend her complaint, purportedly to cure any jurisdictional defects. Such motions usually are granted liberally. Fed. R. Civ. P. 15(a). See Dussouy v. Gulf Coast Investment Corp., 660 F.2d 594 (5th Cir.1981). Yet because the district court found there was no diversity jurisdiction, granting the motion would not have affected the outcome of the case. For the foregoing reasons, the judgment of the district court dismissing the complaint for lack of jurisdiction is affirmed.

__________

 

2. Determining the Amount in Controversy. The diversity jurisdictional statute, 28 U.S.C. § 1332, requires that the “matter in controversy exceeds the sum or value of $75,000, exclusive of interest and costs.” This means diversity of citizenship alone is unavailing; the plaintiff must also be seeking to recover in excess of $75,000.

Since the first Congress, the diversity statute has required a minimum amount in controversy. The Judiciary Act of 1789 set that amount at $500. Congress raised the amount in 1887 to $2000; in 1911 to $3000; in 1958 to $10,000; in 1988 to $50,000, and again in 1996 to $75,000. In part, this jurisdictional amount has been raised in response to broader attempts to abolish diversity jurisdiction. In part, the amount has been raised as Congress attempts to set it “not so high as to convert the Federal courts into courts of big business nor so low as to fritter away their time in the trial of petty controversies.” S. Rep. No. 1830, at 4 (1958), reprinted in 1958 U.S.C.C.A.N. 3099, 3101.

As might be expected, the courts have developed many rules for determining the amount in controversy. Probably the basic rule is that the good faith face of the complaint controls:

The rule governing dismissal for want of jurisdiction in cases brought in the federal courts is that, unless the law gives a different rule, the sum claimed by the plaintiff controls if the claim is apparently made in good faith. It must appear to a legal certainty that the claim is really for less than the jurisdictional amount to justify dismissal.

St. Paul Mercury Indem. Co. v. Red Cab Co., 303 U.S. 283, 288-89, 58 S. Ct. 586, 590, 82 L. Ed. 845, 848 (1938). That general rule is sufficient for purposes of this exercise. This note does not develop rules that surround such questions as when plaintiff’s claimed amount is not in good faith, how to value injunctive or declaratory relief, or when amounts sought in separate claims can be aggregated.[iv]

c. Removal Jurisdiction

Removal jurisdiction is a question of federal subject matter jurisdiction. A defendant in an action filed by plaintiff in a state court may remove the case to federal court, if the federal court would have had original jurisdiction over it. Removal jurisdiction is a one-way street, state court to federal court; removal from federal court to state court does not exist. Also, the federal removal statute, 28 U.S.C. § 1441, allows removal by “the defendant or the defendants.” This means a plaintiff cannot remove a case from state court to federal court, even when the defendant has asserted a counterclaim.

The statute governing removal reads as follows:

(a) Except as otherwise expressly provided by Act of Congress, any civil action brought in a State court of which the district courts of the United States have original jurisdiction, may be removed by the defendant or the defendants, to the district court of the United States for the district and division embracing the place where such action is pending. For purposes of removal under this chapter, the citizenship of defendants sued under fictitious names shall be disregarded.

(b) Any civil action of which the district courts have original jurisdiction founded on a claim or right arising under the Constitution, treaties or laws of the United States shall be removable without regard to the citizenship or residence of the parties. Any other such action shall be removable only if none of the parties in interest properly joined and served as defendants is a citizen of the State in which such action is brought.

(c) Whenever a separate and independent claim or cause of action within the jurisdiction conferred by section 1331 of this title is joined with one or more otherwise nonremovable claims or causes of action, the entire case may be removed and the district court may determine all issues therein, or, in its discretion, may remand all matters in which State law predominates.

* * *

28 U.S.C. § 1441.

__________

As can be seen, the basic removal provision is § 1441(a). Removal is limited in diversity cases by § 1441(b). In addition to these primary provisions, § 1441(c) allows removal when the case includes a “separate and independent claim” based on federal question jurisdiction; most state procedures require some connection, such as being part of the same transaction or occurrence, between claims for them to be joined together in a complaint. The connection required to get the defendants together in state court is usually strong enough to prevent the claims from being separate enough for removal to federal court.[v]

Removal works in this fashion. Suppose a citizen of Connecticut sues a citizen of New York in Connecticut state court on a $150,000 personal injury claim arising from an accident that occurred in Connecticut. The Connecticut court, as a court of general subject matter jurisdiction, could try the case and render a valid judgment. On the other hand, § 1441(a) entitles the defendant to remove the case to federal court (note that if the action were filed in New York state court, § 1441(b) would prohibit removal by the New York defendant). Defendant could file a notice of removal in federal court within 30 days of “receipt by the defendant, through service or otherwise, of a copy of the initial pleading setting forth the claim for relief.” 28 U.S.C. § 1446(b). Upon being notified of the removal, the state court “shall proceed no further” with the case. 28 U.S.C. § 1446(d). A plaintiff who believes the case has been removed improperly, i.e., the federal court lacks subject matter jurisdiction, must bring a motion in the federal court to remand the case to state court. 28 U.S.C. § 1447(c).

The defendant’s attorney might have any number of reasons, good or bad, to want to remove the case to federal court. The attorney might want to take advantage of federal discovery devices. The attorney might believe a federal judge or federal jury would be more favorable to the defendant. The attorney might simply be seeking an advantage if she has more experience in federal court than does plaintiff’s attorney.

d. Supplemental Jurisdiction

Supplemental jurisdiction is discussed in Exercise Six: Joinder and Supplemental Jurisdiction.

C. Personal Jurisdiction

1. Rules of In Personam Jurisdiction

The following paragraphs describe four traditional grounds for jurisdiction over the person of the defendant–in personam jurisdiction–that have achieved general acceptance in American courts.

(a) Consent. A court has personal jurisdiction over a party who consents to the jurisdiction of the court. This consent may be express, as by an admission in open court; implied, as by driving on the roads of a state that has a statute deeming use of the state’s roads to be a submission to jurisdiction; or even inadvertent, as by waiver of the defense for failure to raise it in the first response to the complaint [Fed. R. Civ. P. 12(h)(1). See Exercise Five: Motions to Dismiss and Waiver Under Federal Rule 12].

(b) Domicile. Courts can exercise personal jurisdiction over natural persons who are domiciled within the state even when they are temporarily absent. An analogous rule permits jurisdiction to be asserted over a corporation that is incorporated within the state or has its principal place of business there. Mostly this doctrine is part of the common law, but some states have embodied it in statute.

(c) In‑state Service. Service on a natural person physically present within the territorial jurisdiction of a court secures personal jurisdiction over that defendant. Such “transient jurisdiction” was firmly established prior to Pennoyer v. Neff, 95 U.S. (5 Otto) 714, 24 L.Ed. 565 (1877), and remains good law today. This doctrine came under attack in the 1980s, but was reaffirmed in Burnham v. Superior Court, 495 U.S. 604, 110 S. Ct. 2105, 109 L.Ed.2d 631 (1990).

(d) Contacts with the forum state. After Pennoyer, absent consent or domicile, service on a defendant had to be within the boundaries of the forum state; service outside the state was unavailing. This rigid set of rules became more and more difficult to apply as the twentieth century produced a mobile society, interstate commerce, and corporate defendants. Courts could easily determine whether a natural person had been served within a state, but where was an incorporeal entity such as a corporation located? Under what circumstances was a corporation “present” or “doing business” in a state?

Finally, with its 1945 decision in International Shoe Co. v. Washington, 326 U.S. 310, 66 S. Ct. 154, 90 L.Ed. 95 (1945), the Supreme Court of the United States completely changed the landscape of in personam jurisdiction. International Shoe decided that the due process clause of the United States Constitution allowed service on a defendant outside the state, so long as the defendant “have certain minimum contacts with [the state] such that the maintenance of the suit does not offend ‘traditional notions of fair play and substantial justice.’ “Id. at 316, 66 S. Ct. At 158, 90 L.Ed. at 102. This language became known as the minimum contacts test.

While due process allowed service outside the state, it did not require or implement such service. States had to pass statutes to assert “long-arm” jurisdiction–reaching out to seize a defendant beyond the boundaries of the state. Today all 50 states have long-arm statutes.

When this basis for personal jurisdiction is employed, the court must follow a two-step process. First, the defendant’s act(s) must fall within the ambit of the long-arm statute. That is a question of statutory interpretation. Second, the exercise of jurisdiction under the long-arm statute must comply with the constitutional requirements of due process (the minimum contacts test). Only when both steps are satisfied may the court exercise jurisdiction.

(1) The state long-arm statutes. Over the years since International Shoe, states have enacted two primary types of long-arm statutes. The first type might be called enumerated acts statutes, in which the statute enumerates a list of actions by the defendant that will allow the state to assert jurisdiction over it. The court must decide whether the statute reaches the actions of the defendant. When the statute does not reach the defendant, the court need not consider due process. The second type might be called limits of due process statutes, which simply assert jurisdiction over any action of defendant and cast the entire decision of the appropriateness of jurisdiction on the minimum contacts test. We will discuss both types of long-arm statutes briefly.

The first state to enact a long-arm statute was Illinois in 1955. The drafters of the Illinois statute created an enumerated acts statute, listing four specific actions within the state’s boundaries that would bring a nonresident defendant within the jurisdiction of the state: “transaction of any business,” “commission of a tortious act,” “ownership, use, or possession of any real estate,” or “contracting to insure.” Because Illinois was the first long-arm statute, its language was borrowed, at least in part, by a majority of American states.

Each state drafted its enumerated acts long-arm statute with different provisions, so we reproduce only one example. Uniform Interstate and International Procedure Act § 1.03 was promulgated for adoption by the states by the National Conference of Commissioners on Uniform State Laws in 1962, and it was adopted in many states:

(a) A court may exercise personal jurisdiction over a person, who acts directly or by an agent, as to a [cause of action] [claim for relief] arising from the person’s

(1) transacting any business in this state;

(2) contracting to supply services or things in this state;

(3) causing tortious injury by an act or omission in this state;

(4) causing tortious injury in this state by an act or omission outside this state if he regularly does or solicits business, or engages in any other persistent course of conduct, or derives substantial revenue from goods used or consumed or services rendered, in this state; [or]

(5) having an interest in, using, or possessing real property in this state [; or

(6) contracting to insure any person, property, or risk located within this state at the time of contracting].

(b) When jurisdiction over a person is based solely upon this Section, only a [cause of action] [claim for relief] arising from acts enumerated in this Section may be asserted against him.[vi]

Today, a large majority of American states have enumerated acts long-arm statutes. The situation is complicated, however, by a development in the law that began in Illinois in 1957. The state supreme court, in interpreting the nation’s first long-arm statute, inserted a dictum into its opinion that the statute had a purpose to assert jurisdiction over nonresidents to the full extent permitted by the due process clause. See Nelson v. Miller, 11 Ill. 2d 378, 389, 143 N.E.2d 673, 679 (1957). Even though Illinois continued to interpret its long-arm statute in a limited fashion, many other states’ courts seized on that dictum and held that their limited, enumerated acts long-arm statutes were intended to, and so did, extend to the full limits of due process. Thus, by judicial decision, those courts converted their state statutes from enumerated acts statutes into limits of due process statutes. [vii] Even though such interpretation of detailed statutory language is highly questionable, it induced the withdrawal of Uniform Interstate and International Procedure Act § 1.03 in 1977 as “obsolete.”

The second common type of long-arm statute is the limits of due process statute, which plainly asserts the power of the state over all actions of nonresident defendants that the due process clause, through the minimum contacts test, will allow. The first state to enact such a statute was Rhode Island in 1960. It hewed closely to the International Shoe language by passing a long-arm statute that reaches nonresidents “that shall have the necessary minimum contacts with the state of Rhode Island.” California extended this approach to the ultimate limit in 1969 by passing a statute that reads only “A court of this state may exercise jurisdiction on any basis not inconsistent with the Constitution of this state or of the United States.”

To summarize, long-arm jurisdiction requires the court to consider two steps: the long-arm statute and the due process clause (minimum contacts test). A state with an enumerated acts long-arm statute must first determine whether defendant’s act is encompassed within the language of the statute, and can proceed to the second step only after an affirmative answer. A state that has a limits of due process long-arm statute, or that has interpreted its enumerated acts statute to reach the limits of due process, can proceed directly to the second step.

(2) Constitutional limits on in personam jurisdiction (the minimum contacts test). The long-arm statutes of the states can reach only so far as the Constitution allows. The constitutional limits of long-arm jurisdiction are found in the due process clause.

International Shoe established the test of due process to require that defendant has “certain minimum contacts with [the state] such that the maintenance of the suit does not offend ‘traditional notions of fair play and substantial justice.’ ” 326 U.S. at 316, 66 S. Ct. at 158, 90 L.Ed. at 102. Later cases have interpreted, developed, and expanded this test.

One early case emphasized that these minimum contacts with the state must be voluntary, or in other words, the defendant must “purposely avail” itself of the laws of the forum state. Hanson v. Denckla, 357 U.S. 235, 253, 78 S. Ct. 1228, 1240, 2 L.Ed.2d 1283, 1298 (1958). The contact with the state must have been purposeful. When a retailer sold an allegedly defectively designed automobile to a consumer in New York, who then drove the auto to Oklahoma and had an accident there, jurisdiction was not allowed in Oklahoma even though the retailer could reasonably have foreseen that some of the cars it sold would be driven nationwide. The defendant did not purposefully direct its product/activities toward the forum state. World–Wide Volkswagen Corp. v. Woodson, 444 U.S. 286, 100 S. Ct. 559, 62 L.Ed.2d 490 (1980).

Another decision apparently bifurcated the test into a minimum contacts part and a fair play and substantial justice part. In evaluating the latter portion of the test, the court can consider such things as the burden on the defendant, the plaintiff’s interest, the forum state’s interest, the interstate judicial system’s interest, and the shared policy interests of the several states. Burger King Corp. v. Rudzewicz, 471 U.S. 476-77, 105 S. Ct. 2174, 2184, 85 L.Ed.2d 528, 543 (1985).

Accordingly, a state court may not exercise jurisdiction over a nonresident defendant unless (1) the defendant has had minimum contacts with the state, (2) those contacts were voluntary, and (3) the state is a fair and convenient location for the lawsuit. When the defendant has had no contact with the state, then the state may not assert jurisdiction even if it is a convenient forum for the litigation.

The third requirement, fair play and substantial justice, invites courts to consider a number of convenience factors. One is the probable location of witnesses and evidence. For example, in an action involving injury allegedly caused by a defective product, the state in which the injury occurred is likely to be one in which much of the relevant evidence may be found. The witnesses to the accident, and perhaps to the diagnosis and treatment of the injury, are likely to be there. Physical evidence, such as the product itself, may also be located there.

The forum state’s interest in hearing the lawsuit also has a bearing on whether it can exercise jurisdiction. For example, a state has a particular interest in regulating insurance companies, and hence may hear a suit involving a nonresident insurance company that insured one of its citizens, when perhaps the facts would not justify permitting jurisdiction over an ordinary corporate defendant. See McGee v. International Life Ins. Co., 355 U.S. 220, 78 S. Ct. 199, 2 L.Ed.2d 223 (1957). Thus, businesses that are especially in need of regulation to protect the consumer (insurance, stock brokerage), or companies and individuals that engage in dangerous activities, are subject to broader jurisdiction than other businesses or individuals.

Another factor that courts will consider is the connection between the defendant’s activity in the state and the claim asserted. The question is whether the claim against defendant arises out of its activities within the state. For example, a mail order company may ship a widget to plaintiff in the forum state. Plaintiff is injured when the widget proves defective. Plaintiff’s claim arises from defendant’s contact with the forum state, so this is sometimes called specific jurisdiction. In contrast, a mail order company may ship may widgets into the forum state, but plaintiff is injured when he is struck by one of the company’s delivery vehicles in another state. Plaintiff’s claim does not arise from defendant’s contacts with the forum state, so this is sometimes called general jurisdiction. While due process permits general jurisdiction–power over a defendant based on a claim unrelated to defendant’s contacts with the forum state–many courts are reluctant to assert general jurisdiction over a nonresident defendant and will require a strong showing of continuous and systematic contacts. One example of a court reluctant to assert general jurisdiction can be found in Helicopteros Nacionales de Colombia, S.A. v. Hall, 466 U.S. 408, 104 S. Ct. 1868, 80 L.Ed.2d 404 (1984). This judicial regard for relatedness between defendant’s activity in the state and the claim may arise from concern about the foreseeability of the litigation; a defendant who sells a product within a state should be able to foresee that it might be haled into court to defend lawsuits there based upon that product.

One area of continuing controversy is jurisdiction over a company that puts a product into the stream of commerce, and the stream carries the product into a state where it causes injury. The Supreme Court has split on the issue.. [viii]

An area of emerging controversy is assertion of jurisdiction over a defendant whose contact with the forum state is through the internet. While the Supreme Court has not addressed the issue, lower courts appear to be coalescing around a “sliding scale” test. That was created in Zippo Mfg. Co. v. Zippo Dot Com, Inc., 952 F. Supp. 1119 (W.D. Pa. 1997). The test results in jurisdiction when the defendant’s activity over the internet increases. A passive web site that merely posts information does not create jurisdiction in any one state; a web site that interacts with visitors, as by answering questions submitted, may or may not create jurisdiction; and an active web site that takes orders and sells products into a state does create jurisdiction.

A student of long-arm jurisdiction should note two interesting facts. First, from International Shoe in 1945 until 1980, the trend of the law of personal jurisdiction was decidedly expansionary. The courts appeared to be moving toward a test of pure fairness and convenience, and the long arms of the states became ever longer. With the decision in World-Wide Volkswagen in 1980, the trend halted abruptly. The courts, following the lead of the Supreme Court, have become more wary of jurisdictional assertions, and today closely examine the facts of each individual case to decide whether the particular assertion of jurisdiction satisfies the requirements of due process.

Second, nearly all of the Supreme Court opinions interpreting the requirements of due process over long-arm jurisdiction were decided in a relatively short span of years. The Court became active in the area in 1978 and ceased activity entirely in 1987. Since 1987, the Court has not written an opinion offering additional guidance in the area. At the beginning of the October, 2010, Term of the Supreme Court, it announced the granting of certiorari in two personal jurisdiction cases, both involving the stream of commerce. These cases will be decided during the 2010 Term, and should provide additional, welcome guidance on long-arm jurisdiction.

2. Rules of In Rem Jurisdiction

(a) Types of in rem jurisdiction. Courts may exercise jurisdiction over defendants who own property located within the state. When the action is to determine rights to property, the action is in rem. An action to quiet title or an action to probate an estate are examples of in rem jurisdiction.

When the underlying claim is unrelated to the property, and the property is seized solely to establish jurisdiction in the state, the action is quasi in rem. The location of the property, whether real or personal, in the state, gives the state court power over the action. For example, plaintiff wishes to sue defendant for $100,000 damages incurred in an automobile accident. The accident happened in Florida, but plaintiff wishes to sue in his home state of Illinois. Defendant has never been to Illinois, so that state’s courts cannot assert in personam jurisdiction over her. Defendant’s uncle dies and leaves her a boat docked in Lake Michigan (Chicago docks) worth $35,000. The Illinois court can seize the boat to establish quasi in rem jurisdiction. Should plaintiff prevail in the suit, he can enforce the judgment against the boat, and will have a remaining claim of $65,000 to assert against defendant in another state. Such an assertion of quasi in rem jurisdiction is, however, limited by a standard of reasonableness required by the United States Constitution, as discussed in the next section.

Both of these types of jurisdiction are well recognized by American courts. Recently, courts have sometimes grouped both of these types of jurisdiction together as in rem jurisdiction.

(b) Constitutional limits on in rem jurisdiction. For many years, the International Shoe requirements of fairness and convenience that restrict the assertion of in personam jurisdiction were thought to have no application to in rem and quasi in rem jurisdiction. Under this traditional view, the presence of property within the state could, without more, serve as a sufficient basis for jurisdiction. Thus, if the defendant owned property within State A, that state could take quasi in rem jurisdiction over a case arising from a factually unrelated automobile accident in State B, even if none of the evidence or witnesses were to be found in State A and defendant had never set foot in State A. The theoretical basis for jurisdiction was the state’s power over the property, which would be seized (actually or symbolically) at the commencement of the action. While many states refused jurisdiction in such circumstances, others used their constitutional power to assert such jurisdiction.

In 1977, the Supreme Court swept away the special status of in rem and quasi in rem jurisdiction, holding that they too must be evaluated according to the standard of reasonableness of the minimum contacts test. See Shaffer v. Heitner, 433 U.S. 186, 97 S. Ct. 2569, 53 L.Ed.2d 683 (1977). Consequently, states may continue to classify jurisdiction as in rem or quasi in rem if they wish, but exercises of jurisdiction under those labels are not valid merely because of the existence of property in the state. A nonresident defendant, who is not served in the state, must also have minimum contacts with the state so it is a minimally fair and convenient location for the lawsuit.

3. Personal Jurisdiction of Federal Courts

Even though Congress could probably, within the bounds of the Constitution, provide for exercise of nationwide personal jurisdiction by federal courts, it has not chosen so to extend the federal judicial power. As a general rule, the process (and therefore the personal jurisdiction) of a federal district court may not extend beyond the boundaries of the state in which the court sits, absent a state long-arm statute [Fed. R. Civ. P. 4(k)(1)(A)]. The federal district courts also exercise in rem jurisdiction to the extent it is exercised in the state where the federal court sits, at least when personal jurisdiction in the district cannot otherwise be obtained with reasonable efforts [Fed. R. Civ. P. 4(n)(2).]. Accordingly, federal court personal jurisdiction is generally co‑extensive with state court personal jurisdiction: a federal court sitting within a given state may exercise personal jurisdiction only when a court of that state could do so. A few expansions to this territorial limit have been made by the Federal Rules of Civil Procedure or by statute, but none of these is relevant here.

D. Notice and Opportunity to Be Heard

The requirement of notice and opportunity to be heard is a jurisdictional prerequisite, but is easily satisfied and uncommonly disputed. The brief discussion in I.A., supra, is sufficient for purposes of this exercise.

E. Venue

1. State Courts

While jurisdiction deals with the authority of a court to exercise judicial power, venue deals with the place where that power should be exercised. State venue provisions name a county or a few counties within the state where the suit may be brought. Venue statutes vary widely from state to state, so the individual state’s statute must be consulted.

Typical bases for venue include where the plaintiff resides, where the defendant resides, where the plaintiff or the defendant does business, and where the claim arose. These statutes are written on an abstract assessment of which courts are likely to be convenient to one or both parties. The limiting effect of a venue statute is to preclude the plaintiff from bringing the action in certain counties within the state that are likely to be inconvenient, particularly to the defendant.

2. Federal Courts

As do state venue statutes, the general federal venue statute divides judicial business among the federal courts on a basis of predicted convenience to the parties.

(a) A civil action wherein jurisdiction is founded only on diversity of citizenship may, except as otherwise provided by law, be brought only in (1) a judicial district where any defendant resides, if all defendants reside in the same State, (2) a judicial district in which a substantial part of the events or omissions giving rise to the claim occurred, or a substantial part of property that is the subject of the action is situated, or (3) a judicial district in which the defendants are subject to personal jurisdiction at the time the action is commenced, if there is no district in which the action may otherwise be brought.

(b) A civil action wherein jurisdiction is not founded solely on diversity of citizenship may, except as otherwise provided by law, be brought only in (1) a judicial district where any defendant resides, if all defendants reside in the same State, (2) a judicial district in which a substantial part of the events or omissions giving rise to the claim occurred, or a substantial part of property that is the subject of the action is situated, or (3) a judicial district in which any defendant may be found, if there is no district in which the action may otherwise be brought.

(c) For purposes of venue under this chapter, a defendant that is a corporation shall be deemed to reside in any judicial district in which it is subject to personal jurisdiction at the time the action is commenced. In a State which has more than one judicial district and in which a defendant that is a corporation is subject to personal jurisdiction at the time an action is commenced, such corporation shall be deemed to reside in any district in that State within which its contacts would be sufficient to subject it to personal jurisdiction if that district were a separate State, and, if there is no such district, the corporation shall be deemed to reside in the district within which it has the most significant contacts.

(d) An alien may be sued in any district.

* * *

28 U.S.C. § 1391.

__________

 

As can be seen, § 1391(a) governs venue in diversity cases, and § 1391(b) governs venue in all other–typically federal question–cases. Historically, these two subsections were substantially different. For example, at one time, the district where all plaintiffs resided was an appropriate venue in diversity cases, but not in other cases. With the most recent revision of the venue statute in 1990, the two provisions are largely identical, and thus mostly redundant. Congress could revisit the statute and meld the two sections, but as with many areas of judicial administration, simply has not got around to it.

Accordingly, 28 U.S.C. §§ 1391(a)(1) and (b)(1) in all cases allow venue to be laid in a district where a defendant resides so long as all defendants reside in the same state. The statute does not require all defendants reside in the same federal district. Many states include more than one federal district. All defendants must reside in the same state, not the same district. Federal courts treat residence in the venue statute as identical to citizenship, which in turn is determined by domicile. See I.B.2.b., supra. When plaintiff sues multiple defendants, and not all reside in the same state, this venue possibility is unavailable.

28 U.S.C. §§ 1391(a)(2) and (b)(2) in all cases also allow venue to be laid in a district where “a substantial part of the events or omissions giving rise to the claim occurred.” This second option may be only one state, or it may include several states. At one time, the statutory language for this option provided “where the claim arose.” This was properly interpreted to mean a single state, and it produced much litigation when the claim arose in activities covering multiple states. Congress amended the statute in 1990 so that every state in which a substantial part of the claim arose is a proper venue.

The third options in §§ 1391(a) and 1391(b) are alike in that both are fallback provisions. They are available only when “there is no other district in which the action may otherwise be brought.” That means all defendants do not reside in the same state and a substantial part of the claim did not arise in any district. This situation may occur when the claim arose in a foreign country. Despite this similarity, each of the two provisions does differ. For diversity cases, § 1391(a)(3) venue is proper in any district where any defendant is “subject to personal jurisdiction at the time the action is commenced.” For cases other than diversity, § 1391(b)(3) venue is proper in any district in which “any defendant may be found.” These two differing phrases are probably the result of legislative inadvertence in the Congressional drafting process. Accordingly, a court will likely interpret either as requiring only that a court have personal jurisdiction over a defendant. On the other hand, the language of the two sections is different and could lead to differing interpretations. For example, § 1391(b)(3) requires a district where “defendant may be found.” This could mean where a defendant is present for in-state service. In differing language, § 1391(a)(3) requires personal jurisdiction “at the time the action is commenced.” This could mean venue does not lie when defendant was not subject to jurisdiction when the action was commenced by filing with the court [Fed. R. Civ. P. 3], even though defendant is later subject to jurisdiction in the state.

The venue provisions over certain types of defendants or actions are broader. A corporation is deemed by § 1391(c) to reside in any district in which it is subject to personal jurisdiction, which will almost certainly broaden the proper venues under both § 1391(a)(1) and §1391(b)(1). Venue in a suit against an alien may be laid in any district by 28 U.S.C. § 1391(d). In addition to the proper venues in § 1391, many federal statutes have individual venue provisions, e.g., 28 U.S.C. § 1396 (internal revenue taxes); 28 U.S.C. § 1400 (patent or copyright laws). For this exercise, we will limit our consideration to the general federal venue statute, 28 U.S.C. § 1391.

When a plaintiff lays venue in a wrong federal district, the defendant may waive objection to venue, either intentionally or inadvertently [Fed. R. Civ. P. 12(h)(1). See Exercise Five: Motions to Dismiss and Waiver Under Federal Rule 12]. When the defendant does object to venue, the court may either dismiss under Fed. R. Civ. P. 12(b)(3) for improper venue or transfer the action pursuant to the authority of 28 U.S.C. § 1406(a) to a district “in which it could have been brought.”

3. Forum Non Conveniens

Sometimes a state trial court may decline to hear a case even when jurisdiction and venue are proper. Under the doctrine of forum non conveniens, the court has discretion to dismiss an action if the forum is so inconvenient that justice requires the action be brought elsewhere. A state court can only retain the action or dismiss; it cannot transfer the action to another state.

A federal court has the additional option of transfer of the case to a more convenient federal district: “For the convenience of parties and witnesses, in the interest of justice, a district court may transfer any civil action to any other district or division where it might have been brought.” 28 U.S.C. § 1404(a). In deciding whether to grant transfer under § 1404(a), federal courts generally take into account the same factors as do state courts in ruling on forum non conveniens motions, including availability of evidence, possibility of view, location of the parties, and location of witnesses.

A federal court may also dismiss on forum non conveniens when the more convenient court is in a foreign country. In making its decision whether to retain the case or to dismiss and hence require plaintiff to recommence in the foreign court, the federal court will consider “private interest factors,” including convenience to parties and witnesses, and “public interest factors,” including familiarity with the governing law. See Piper Aircraft Co. v. Reyno, 454 U.S. 235, 102 S. Ct. 252, 70 L.Ed.2d 419 (1981).

II. QUESTIONS ON JURISDICTION

This section contains questions for you to answer to test and strengthen your knowledge of the law of jurisdiction. Use your scrolling feature so that the screen shows only the question. Answer the question, then scroll down to compare your answer to the authors’ answer. P represents plaintiff and D represents defendant.

A. Subject Matter Jurisdiction

1. Original Jurisdiction

Q–1. P1, an individual, and P2, a corporation, file a joint complaint in state X’s court of general jurisdiction against defendants D1, D2, and D3. The complaint alleges several counts: violation of federal civil rights law, breach of contract, wrongful interference with business, and trespass to land. Is there subject matter jurisdiction?

Answer to Q–1

Yes. A court of general subject matter jurisdiction has power to hear cases involving all types of parties and all types of claims. There is no jurisdiction problem here, although there may be a joinder problem (depending on the state’s joinder provisions).

The fact that one of the theories presents a federal question may allow, but does not require, the case to be brought in federal court. This would be an example of concurrent jurisdiction (although some federal questions, such as bankruptcy, are exclusive federal jurisdiction).

 

Note: The remaining questions in this section place the action in federal court.

 

 

 

Q–2. P enters a contract with D Mint to provide 1000 commemorative coins for a festival P is promoting. D Mint is later informed by a Treasury agent that the coin would be too close to U.S. currency, and D Mint refuses to perform the contract. P pleads breach of contract, and that the defense of violation of federal coinage laws is ineffective. Is this a federal question?

 

 

 

Answer to Q–2.

No. A federal question cannot arise in defense. See Louisville & Nashville R.R. v. Mottley, 211 U.S. 149, 29 S. Ct. 42, 53 L.Ed. 126 (1908), reproduced in Exercise One, part II. This question presents a variation on the same theme: the federal question is raised in an anticipated defense, which the defendant may or may not choose to assert. The federal question must arise in the well‑pleaded complaint.

 

 

 

 

Q–3. P is employed by D Corporation, which operates nationally. On P’s 50th birthday, a supervisor fires her because “we do not want people above a certain age working for us.” P sues in federal court for breach of the employment contract. Is this a federal question?

 

 

 

Answer to Q–3.

No. Although plaintiff would appear to be able to assert a claim under the federal Age Discrimination in Employment Act on these facts, she has not done so. She pleaded only breach of contract, a state law theory. As a general proposition, plaintiff is master of her own complaint. This case is not within 28 U.S.C. § 1331.

 

 

 

 

 

Q–4. P purchases stock from D in reliance on representations that prove false. P sues D for violation of § 10(b) of the Securities Exchange Act of 1934, for violation of the state anti‑fraud securities statute, and for common law fraud. Is this a federal question?

 

 

 

 

Answer to Q–4.

Yes. P’s claim arises under a federal statute, the Securities Exchange Act of 1934. P’s other theories of recovery, the state statute and common law fraud, while state law theories, can also be heard in federal court under the doctrine of supplemental jurisdiction. 28 U.S.C. § 1367. See Exercise Six: Joinder and Supplemental Jurisdiction.

 

 

 

 

Q–5. P, a citizen of California, sues D, a citizen of Iowa. The damages sought exceed the jurisdictional amount required. Is there diversity jurisdiction?

Answer to Q–5.

 

 

 

 

Yes. Diversity jurisdiction exists because the suit is between citizens of different states, and the minimum amount in controversy is satisfied. 28 U.S.C. § 1332(a)(1).

 

 

 

 

 

Q–6. P, a citizen of France, sues D, a citizen of Iowa, for damages exceeding the jurisdictional amount. Is there diversity jurisdiction?

 

 

 

 

Answer to Q–6.

Yes. Diversity jurisdiction exists because the amount in controversy is sufficient and the suit is between a citizen of a state and a citizen or subject of a foreign state. 28 U.S.C. § 1332(a)(2). This is commonly called alienage jurisdiction.

 

 

 

 

 

Q–7. Part 1. P, a citizen of Germany, sues D, a citizen of Great Britain, for damages exceeding the jurisdictional amount. Is there diversity jurisdiction?

Part 2. Assume Congress amends 28 U.S.C. § 1332(a) to include a new provision granting the district courts jurisdiction over civil actions between aliens. Does this confer federal question jurisdiction in part 1?

 

 

 

 

Answer to Q–7. Part 1.

No. There is no diversity jurisdiction because the suit is between citizens of two foreign states and hence does not fall within 28 U.S.C. § 1332(a)(2).

Part 2. No. Even though the suit between two aliens would, in this hypothetical, fall within 28 U.S.C. § 1332, the statute would be unconstitutional. Article III, § 2 of the United States Constitution (reproduced in I.B.2, supra) grants the federal courts authority over only specific classes of cases, and a suit between two aliens is not included.

 

Note that in Q–6 and Q–7, the interpretation of the statute is consistent with the goal of protecting out‑of‑state litigants from actual or perceived bias that they might encounter if forced to sue in state court. Were a citizen of California or of France forced to sue an Iowa citizen in Iowa state court, there might be a real or perceived danger of bias in favor of the Iowa citizen. On the other hand, the state court would presumably be neutral in a suit between two aliens. As is often the case, the somewhat ambiguous language of the statute (conferring jurisdiction upon suits “between … citizens of a State and citizens or subjects of a foreign state”) can be illuminated by reference to its underlying purpose.

Admittedly, the hodge podge of rules about diversity jurisdiction cannot always be explained in terms of the supposed goal of preventing local bias. For example, a citizen of Virginia may institute a diversity suit against a citizen of Georgia in federal court in Virginia, despite the fact that there is no a priori reason to believe that a citizen of Virginia would need to fear bias from a home state court.

 

Q–8. P, a citizen of Texas, sues D, a citizen of the United States who is domiciled in Arkansas, for an amount in excess of the jurisdictional amount. Is there diversity jurisdiction?

 

 

 

 

 

Answer to Q–8.

Yes. A natural person who is a citizen of the United States is also a citizen of the state in which that person is domiciled. See Scoggins v. Pollock, in I.B.2.a.1, supra. Therefore, diversity of citizenship jurisdiction exists under 28 U.S.C. § 1332(a)(1).

 

 

 

 

 

Q–9. P, a citizen of Illinois, sues D, a citizen of Mexico who has been admitted to the United States for permanent residence and is domiciled in Illinois, for an amount in excess of the jurisdictional amount. Is there diversity jurisdiction?

 

 

 

Answer to Q–9.

No. Although 28 U.S.C. § 1332(a)(2) would appear to grant jurisdiction in this situation since the suit is between “citizens of a State and citizens or subjects of a foreign state,” there is no diversity of citizenship because the same statute also provides “[A]n alien admitted to the United States for permanent residence shall be deemed a citizen of the State in which such alien is domiciled.” The intent of this provision is to restrict diversity jurisdiction. Accordingly, D will be deemed a citizen of Illinois, and since P is a citizen of Illinois, there is no diversity.

 

 

 

 

 

 

Q–10. Part 1. P, a citizen of Florida, sues D, an American citizen who moved to Florida from Michigan three months prior to the commencement of the action. D was born and reared in Michigan, and voted there in the last election, but now has an abode in Florida and intends to remain there permanently. The amount in controversy exceeds the jurisdictional amount. Is there diversity jurisdiction?

Part 2. Assume the same facts as Part 1, except D moves to Florida three months after commencement of the action. Is there diversity jurisdiction?

Part 3. P and D were both citizens of Michigan at the time the claim arose. P moved to Florida and became a citizen there for the avowed purpose of creating diversity jurisdiction, then commenced the action. Is there diversity jurisdiction?

 

 

 

 

 

 

 

Answer to Q–10. Part 1.

No. Since D has physically moved to Florida and intends to remain there permanently, D has become domiciled in Florida. D is consequently a citizen of Florida.

Part 2. Yes. Citizenship is determined as of the day of the commencement of the action. Diversity is not destroyed by later actions of the parties.

Part 3. Yes. Diversity is determined by the citizenship of the parties as of the day of commencement. P’s motive in becoming a Florida citizen is not relevant. See Charles A. Wright & Mary Kay Kane, The Law of Federal Courts § 28 (6th ed. 2002). D might challenge P’s intent to remain in the new state indefinitely, as was done in Baker v. Keck [Exercise One, part III, and part I.B.2.b.1, supra], but this question states P became a citizen of Florida.

 

 

 

 

 

 

 

Q–11. P, a citizen of Ohio, sues D, an American citizen who was born and reared in Ohio, but who left two months ago to work for the Peace Corps in Costa Rica. D does not know where she will live after she finishes her two‑year stint in the Peace Corps, but she has a definite intention not to live in either Ohio or Costa Rica. The amount in controversy exceeds the jurisdictional amount. Is there diversity jurisdiction?

 

 

 

 

 

Answer to Q–11.

No. Since D has not acquired a new domicile, she is deemed to have retained her old domicile in Ohio despite her intention not to return there. See Scoggins v. Pollock [part I.B.2.b.1, supra].

 

 

 

 

 

Q–12. P, a citizen of Oregon, sues D Corporation, which is incorporated in Delaware and has its principal place of business in Oregon, for an amount exceeding the jurisdictional amount. Is there diversity jurisdiction?

 

 

 

 

 

 

Answer to Q–12.

No. The corporation has dual citizenship: Delaware and Oregon. 28 U.S.C. § 1332(c)(1) provides “[A] corporation shall be deemed to be a citizen of any State by which it has been incorporated and of the State where it has its principal place of business.” Therefore, the plaintiff and defendant are citizens of the same state–Oregon–and there is no diversity jurisdiction.

Prior to the enactment of 28 U.S.C. § 1332(c) in 1958, a corporation was a citizen only of the state in which it was incorporated. Therefore, a corporation owned and operated in Oregon by Oregonians could litigate its local disputes in federal court if it were formally incorporated in another state, even though it would have no legitimate reason to fear prejudice at the hands of Oregon state courts. Section 1332(c) was amended to correct this anomaly.

 

 

 

 

 

Q–13. P, a citizen of North Carolina, sues D, a citizen of New York, for $200,000 for personal injuries arising out of an automobile accident in Pennsylvania. Suit is brought in federal district court in North Carolina. D has never been to North Carolina and has no connection with it except for this suit. Is there diversity jurisdiction?

 

 

 

 

 

 

Answer to Q–13.

Yes. Diversity jurisdiction is a form of subject matter jurisdiction. It is based on the theory that a federal court is particularly competent, because of its neutrality, to decide cases involving citizens of different states. Personal jurisdiction is a separate jurisdictional requirement based upon the connection of the defendant to the forum state. Requirements of personal jurisdiction are designed to ensure that the suit is brought in a state that has power over the defendant and is a geographic location that is reasonably fair and convenient. Venue is also a separate requirement, dealing with convenience of the location of the action. Diversity jurisdiction exists here. The question does not ask about personal jurisdiction or venue; objection to either would likely result in dismissal.

 

 

 

 

 

 

Q–14. A, a citizen of Nebraska, sues D1, a citizen of Washington, and D2, a citizen of Tennessee, for a claim that arose at a luau while all three parties were vacationing in Hawaii. The claim exceeds the jurisdictional amount. P files the action in the District of Hawaii. Is there diversity jurisdiction?

 

 

 

 

 

Answer to Q–14.

Yes. Diversity of citizenship exists between the parties. The fact that the suit was brought in Hawaii does not deprive the court of diversity jurisdiction. [Personal jurisdiction and venue are also likely proper here.]

This rule has been criticized on the ground a local state court would not likely be biased in a suit between two noncitizens. For example, there is no reason to believe the state court of Hawaii would favor one party or the other in a suit between a plaintiff from Nebraska and defendants from Washington and Tennessee. Hence, no reason exists to confer federal diversity jurisdiction upon that case. The purpose of diversity jurisdiction would seem to be served adequately by a rule that provided a federal forum for suits brought in a state in which one of the parties was domiciled; however, 28 U.S.C. § 1332 confers diversity jurisdiction upon all federal courts, regardless of whether any of the parties is a citizen of the state in which the suit has been brought. When one federal court has diversity jurisdiction, then all federal courts have diversity jurisdiction.

 

 

 

 

 

Q–15. P, a citizen of Delaware, sues D1, a citizen of New Jersey, and D Corporation, which is incorporated in Delaware and has its principal place of business in New Jersey, in the District of New Jersey. The amount in controversy exceeds the jurisdictional amount. Is there diversity jurisdiction?

 

 

 

 

 

Answer to Q–15.

No. “Complete diversity” does not exist. There is no diversity of citizenship between P and D Corporation, since both are citizens of Delaware. There is diversity between P and D1. The issue, then, is whether minimal diversity, i.e., between one plaintiff and one defendant, is sufficient to establish jurisdiction under § 1332. In Strawbridge v. Curtiss, 7 U.S. (3 Cranch) 267, 2 L.Ed. 435 (1806), Chief Justice John Marshall interpreted the language of a predecessor statute to § 1332(a)(1) to mean diversity of citizenship must exist between each plaintiff and each defendant. When one plaintiff and one defendant are citizens of the same state, the federal court has no jurisdiction, even if there is diversity among all other parties.

This rule of complete diversity has been criticized because it excludes federal jurisdiction in circumstances in which there may be a genuine danger of local prejudice. For example, P might have a genuine reason to fear prejudice in state court in New Jersey. Chief Justice Marshall himself is said to have expressed doubts about the wisdom of Strawbridge, but the rule of complete diversity remains good law. We do know today that complete diversity is required only by the statute, and not by Article III, § 2 of the Constitution. See State Farm Fire & Cas. Co. v. Tashire, 386 U.S. 523, 87 S. Ct. 1199, 18 L.Ed.2d 270 (1967).

 

2. Removal Jurisdiction

 

Q–16. P1, a citizen of New Mexico, and P2, a citizen of Colorado, sue D1, a citizen of Utah, and D2, a citizen of California, in Arizona state court. The claim arises under Arizona state law. The amount in controversy exceeds the jurisdictional amount. May the defendants remove the action to federal court?

 

 

 

 

Answer to Q–16.

Yes. The parties are of diverse citizenship and the requirements of 28 U.S.C. § 1441(a) have been met. The fact that the Arizona court has concurrent jurisdiction does not prevent removal of the case to federal court.

 

 

 

 

 

Q–17. Part 1. P, a citizen of Kentucky, sues D, a citizen of Indiana, in state court in the state of Kentucky. The claim arises under state law and the amount in controversy is $50,000. May D remove the case to federal court?

Part 2. Assume instead that the amount in controversy is $75,000. May D remove the case to federal court?

 

 

 

 

 

Answer to Q–17. Part 1.

No. An action is removable only if the federal district court has original jurisdiction. 28 U.S.C. § 1441(a). This action could not have been brought in federal court because the amount in controversy does not satisfy the jurisdictional amount.

Part 2. No, for the same reason. The jurisdictional amount is satisfied only when “the matter in controversy exceeds the sum or value of $75,000***.” 28 U.S.C. § 1332(a).

 

 

 

 

 

 

Q–18. Part 1. P sues D in state court in Connecticut. The claim arises under state law and the amount in controversy is $200,000. P was born and raised in Vermont, but moved to Connecticut shortly before the commencement of the action and intends to remain there permanently. D is domiciled in Connecticut. May D remove the action to federal court?

Part 2. P remains domiciled in Vermont until after she commences the action in Connecticut state court. D is domiciled in Connecticut. May D remove the action to federal court?

 

 

 

 

 

Answer to Q–18. Part 1.

No. The district court does not have original jurisdiction because both parties are citizens of Connecticut. P has acquired a domicile in Connecticut because the two elements of physical presence and intent to remain indefinitely are satisfied.

Note that the technically correct answer to the question posed is yes, D may remove the case to federal court because a notice of removal operates automatically to remove the case from state to federal court. 28 U.S.C. § 1446(d). Since the case would not have properly been removed, however, plaintiff will succeed in a motion to remand the case to state court. 28 U.S.C. § 1447(c).

Part 2. No. Although the district court would have original jurisdiction over the action since the parties are of diverse citizenship, and plaintiff could have sued originally in federal court, the removal statute does not allow a defendant who is a citizen of the state in which the action was brought to remove. 28 U.S.C. § 1441(b). The rationale is that a defendant who is a citizen of the state need not fear local prejudice.

 

 

 

 

 

 

Q–19. P Corporation, which is incorporated and has its principal place of business in Massachusetts, sues X Corporation, which is incorporated and has its principal place of business in California, and Y Corporation, which is incorporated in Delaware and has its principal place of business in New York. The action is commenced in state court in New York. The claim is breach of a contract in which X Corporation promised to deliver widgets to P Corporation and Y Corporation issued a surety bond for performance. The amount in controversy is one million dollars and the claim arises under state law. May the defendants remove the case to federal court?

 

 

 

 

 

Answer to Q–19.

No. Although the parties are of diverse citizenship and the district court would have original jurisdiction, Y Corporation is a citizen of New York because it has its principal place of business there [28 U.S.C. § 1332(c)(1) ], so as in Q–18, part 2, the action is not removable because one of the defendants is a citizen of the state in which the action was brought.

The claim against the California corporation does not fit within 28 U.S.C. § 1441(c) for two reasons: no federal question is involved, and there is no “separate and independent claim or cause of action.” See Charles A. Wright & Mary Kay Kane, The Law of Federal Courts § 39 (6th ed. 2002).

 

 

 

 

 

Q–20. P, a citizen of Alabama, sues D, a citizen of Louisiana, in state court of Louisiana. The action arises under a federal statute that confers concurrent jurisdiction upon state and federal courts. The amount in controversy exceeds the jurisdictional amount. May D remove the action to federal court?

 

 

 

 

 

Answer to Q–20.

Yes. The subject matter jurisdiction of the federal court is based upon the existence of a federal question and therefore the action is removable “without regard to the citizenship or residence of the parties.” 28 U.S.C. § 1441(b). Federal question cases are treated differently from diversity cases because federal question jurisdiction is not based upon fear of prejudice in state courts against non‑citizens, but upon the special competency of the federal courts to decide questions of federal law.

B. Personal Jurisdiction

 

Q–21. Part 1. Defendant of Nebraska went to Colorado for a picnic. She stayed only a few hours. While in Colorado, she was served with process from a Colorado state court. The action arose from an automobile accident that occurred in Wyoming. Does Colorado have personal jurisdiction in the action?

Part 2. Same facts as Part 1, except that the defendant is a corporation. Its President goes to Colorado for a short personal errand and is served with process. Does Colorado have jurisdiction over the action?

 

 

 

 

 

Answer to Q–21. Part 1.

Yes. Defendant was served in the state. Assertion of “transient” or “territorial” jurisdiction is of long standing, and has been upheld against constitutional attack even where defendant had no connection with the forum other than service in the state. Burnham v. Superior Court, 495 U.S. 604, 110 S. Ct. 2105, 109 L.Ed.2d 631 (1990).

Part 2. No. Transient jurisdiction normally applies only when the defendant is a natural person present within the state. A corporation is an artificial person. No other basis for personal jurisdiction exists here.

 

 

 

 

 

 

Q–22. D Corporation was incorporated in Nevada solely to obtain the benefits of a favorable corporation law. D Corporation’s only place of business is in Texas. Other than incorporation, it has no contacts at all with Nevada: no property, no business activity, no shareholders. P, a Texas resident, sues D Corporation in Nevada state court on an automobile accident that occurred in Texas. May Nevada exercise personal jurisdiction over D Corporation?

 

 

 

 

 

Answer to Q–22.

Yes. Incorporation in a state gives the state power to subject the corporation to its courts. This is akin to service on a natural person within a state.

 

 

 

 

 

 

Q–23. D Corporation is incorporated and has its principal place of business in state A. It ships 2000 cases of beer a year to X, a wholesaler in state B. It does no other business in state B. P, a management consultant in state B, sues D Corporation in state court of state B claiming failure to pay for services he performed for D Corporation in state A. May state B exercise personal jurisdiction in the case of P v. D Corporation?

 

 

 

 

 

 

Answer to Q–23.

Probably no. First, even though D Corporation is transacting business in state B, that business has no connection with the claim. The long-arm statutes of many states require that the claim arise out of the events that take place in the state. [See, e.g., Uniform Interstate and International Procedure Act § 1.03(b) in part I.C.1, supra].

Second, even in those states that have long-arm statutes that reach to the limits of due process–whether by specific provision or court interpretation–D Corporation would not likely be found to have sufficient minimum contacts such as not to offend traditional notions of fair play and substantial justice in state B. This would be an attempted assertion of general jurisdiction by state B (the claim does not arise from the contacts), and courts require a high level of continuous and systematic business to establish minimum contacts with the state in such a case.

 

 

 

 

 

 

Q–24. Under a long‑arm statute, a state court in state X can exercise personal jurisdiction over defendant based on defendant’s minimum contacts with state X. Does a federal court sitting in state X also have personal jurisdiction over defendant?

 

 

 

 

 

 

Answer to Q–24.

Yes. Because Fed. R. Civ. P. 4(k)(1)(A) incorporates state long-arm statutes, a federal court may exercise personal jurisdiction whenever the state in which it is located would have personal jurisdiction. [A federal court may also exercise personal jurisdiction under Fed. R. Civ. P. 4(k) in a few situations when the state court could not, but these are not relevant here.]

 

 

 

 

 

 

Q–25. Part 1. D Corporation is incorporated and has its only place of business in state A. It contracts with P, a resident of state B, to ship 100 cases of whiskey from state A to state B. D Corporation fails to ship the whiskey, and consequently, P sues D Corporation in state court in B. May state B exercise personal jurisdiction over D Corporation?

Part 2. Same facts as Part 1, with the additional facts that D Corporation has shipped whiskey to P in state B for several years, P has always paid for those shipments with checks drawn on a bank in state B, and P has negotiated prices and other contract terms with D Corporation over the telephone and by mail from P’s office in state B. May state B exercise personal jurisdiction over D Corporation?

 

 

 

 

 

Answer to Q–25. Part 1.

No. The long-arm statute is likely to be satisfied. For example, Uniform Interstate and International Procedure Act § 1.03(a)(2), in part I.C.3, supra, provides jurisdiction when the claim arises from defendant’s “contracting to supply services or things in this state.” Second, however, the defendant must have minimum contacts with the state so that exercise of jurisdiction over it would not offend traditional notions of fair play and substantial justice. On the facts here, D Corporation has no contact with state B other than a single, unperformed contract. A court will conclude the exercise of jurisdiction would violate due process.

Part 2. Yes. As in Part 1, the long-arm statute is satisfied. The additional facts clearly supply sufficient minimum contacts of D Corporation with state B for the court to exercise jurisdiction.

 

 

 

 

 

 

 

Q–26. D Corporation, located solely in Michigan, has supplied component parts to Acme Manufacturing Co., located in North Carolina, for several years. Acme is D Corporation’s primary customer. Acme uses the parts and ships the final product nationwide. One of the parts fails and P is injured in New York. Does New York have personal jurisdiction over D Corporation?

 

 

 

 

 

Answer to Q–26.

Maybe. The first question is whether the long-arm statute reaches defendant’s actions, so we would look at the New York statute. If we assume the New York statute is similar to Uniform Interstate and International Procedure Act § 1.03(a)(4), in part I.C.3, supra, D Corporation did cause “tortious injury in this state by an act or omission outside this state;” the additional requirement is that D Corporation “derives substantial revenue from goods or services used or consumed in this state.” This element will be more difficult to satisfy and will likely require production of a record to show the revenues.

The due process step may or may not be satisfied. D Corporation has placed its product into the stream of commerce, perhaps knowing that some portion will flow into New York. There are no other known contacts. The plurality opinion in Asahi Metal Industry Co. v. Superior Court, 480 U.S. 102, 107 S. Ct. 1026, 94 L.Ed.2d 92 (1987), disapproves the stream of commerce theory as a basis for personal jurisdiction. Two concurring opinions, commanding five votes, appear to approve the stream of commerce theory, but agree on dismissal for other reasons. Accordingly, the constitutional acceptability of the stream of commerce theory is in doubt.

C. Notice and Opportunity to Be Heard

 

Q–27. P landlord commenced an action to evict D tenant pursuant to state unlawful detainer law for failure to pay rent. Service was made on D by tacking the summons and complaint to D’s apartment door, even though such notices had sometimes in the past disappeared from other tenants’ doors. Is this constitutionally sufficient notice to D?

 

 

 

 

 

Answer to Q–27.

No. Due process requires that P choose a method of notice that is reasonably calculated to give actual notice, “reasonably certain to inform those affected.” Mullane v. Central Hanover Bank & Trust Co., 339 U.S. 306, 315, 70 S. Ct. 652, 657, 94 L.Ed. 865, 874 (1950). Here P was aware of D’s location and chose to notify by posting on a door to a common area, even though other such notices had disappeared. The Supreme Court held such notice is constitutionally inadequate in Greene v. Lindsey, 456 U.S. 444, 102 S. Ct. 1874, 72 L.Ed.2d 249 (1982).

 

 

 

 

 

 

Q–28. State X’s garnishment statute allows seizure when plaintiff alleges defendant is about to waste property in which plaintiff has an ownership interest. Plaintiff must file a detailed affidavit of the facts and post a bond. The garnishment order must be signed by a judge. Plaintiff is allowed a hearing on the seizure “no later than two weeks after the sheriff takes possession of the property.” Does this procedure provide a constitutional opportunity to be heard?

 

 

 

 

 

 

Answer to Q–28.

Maybe. The Supreme Court’s earlier pre‑judgment seizure cases seem to require four criteria be met for a constitutional seizure: 1) plaintiff must file a particularized affidavit, 2) plaintiff must post a bond, 3) a judge must approve the seizure, and 4) defendant must receive a right to an early hearing. The first three appear to be met. The question is whether a hearing “no later than two weeks” after the seizure is a “prompt post deprivation hearing.” The Court did not specify a time limit. The Court’s latest decision, Connecticut v. Doehr, 501 U.S. 1, 111 S. Ct. 2105, 115 L.Ed.2d 1, (1991), turns toward a balancing test to evaluate a pre-judgment seizure. The court is to balance the interest of the party from whom the property is seized, the interest of the party seizing the property, and the risk of erroneous deprivation. To apply such a balancing test in this question, we would need additional facts.

D. Venue

 

Q–29. P, a resident of South Dakota, sues D, a resident of Kansas, in federal district court in Nebraska on a claim based upon an automobile accident in Nebraska. Jurisdiction is based solely on diversity of citizenship. Is venue properly laid in Nebraska?

 

 

 

 

 

Answer to Q–29.

Yes. A substantial part of the events or omissions giving rise to the claim–indeed the whole claim–arose in Nebraska, which is a permissible venue. 28 U.S.C. § 1391(a)(2). Another permissible venue would have been Kansas, where defendant lives. 28 U.S.C. § 1391(a)(1).

 

 

 

 

 

 

Q–30. Part 1. P, a resident of Illinois, sues D, a resident of Ohio, and D Corporation, which is incorporated and has its principal place of business in Indiana. D Corporation regularly does business and so is subject to personal jurisdiction in Illinois, Ohio, Indiana, and Kentucky. The sole basis for jurisdiction is diversity of citizenship. The claim arises from an automobile accident in California. Is venue properly laid in federal district court in Ohio?

Part 2. Could venue properly be laid in federal district court in Illinois?

Part 3. Could venue properly be laid in federal district court in Indiana?

Part 4. Would your answer to any of the above parts of this question be different if federal subject matter jurisdiction were based on a federal question instead of diversity of citizenship?

Answer to Q–30, Part 1.

Yes. Both defendants reside in Ohio, so venue is proper under 28 U.S.C. § 1391(a)(1). D Corporation is deemed a resident of Ohio under § 1391(c) because it is subject to personal jurisdiction there when the action is commenced.

Part 2. No. The residence of the plaintiff is not a permissible venue under § 1391(a). Both defendants do not reside in Illinois, so § 1391(a)(1) does not apply, and no part of the claim arose there, so § 1391(a)(2) does not apply.

An argument that § 1391(a)(3) applies, allowing venue in a district “in which any defendant is subject to personal jurisdiction,” would fail. As a fallback provision, § 1391(a)(3) applies only when “there is no other district in which the action may otherwise be brought.” As can be seen in part 1, the action may be brought in Ohio under § 1391(a)(1). Also, the action may be brought in California under § 1391(a)(2).

Part 3. No. Venue may be laid, pursuant to § 1391(a)(1), in a district where “any defendant resides, if all defendants reside in the same State.” Defendant D Corporation resides in Indiana, but defendant D does not. No part of the claim arose in Indiana. Since venue could be laid in either Ohio or California, the fallback provision cannot be used.

Part 4. No. The two venue statutes differ only in their third subsections. The “catch‑all” federal question venue subsection, § 1391(b)(3), is worded differently from the “catch‑all” diversity venue subsection, § 1391(a)(3), but both are similar in that they apply only when venue cannot properly be laid in another district. From parts 1 and 2, both Ohio and California would be proper venues.

III. COMPUTER EXERCISES

A. CALI CIV 03: Jurisdiction and Venue

This computer-assisted exercise is based on the following fact scenario. Study these facts carefully. After the facts, in part III.A.3, infra, you will find several questions raising jurisdictional questions about the facts. The computer will ask the same questions, including what reasons you give for your answers, so we advise you to consider these questions carefully and reach your best answers before you go to the computer.

This is a lengthy computer-assisted exercise. Our estimated completion time is two hours. We recommend that you attempt to complete this exercise in one sitting because your answers to earlier questions will be saved by the computer and used in later questions.

1. Facts

You are a lawyer working for a firm in the state of Fraser. One of the firm’s partners asks you to do some research on a personal injury action. Pam Pedestrian is a local resident who was involved in an accident in the state of Elliott while on summer vacation there. A truck driven by a servant of D Corporation collided with a car driven by David Driver, causing Driver’s car to swerve onto the sidewalk and strike Pedestrian. The partner wants to sue both D Corporation and David Driver, and asks you to identify states where the action can be brought without being dismissed for lack of jurisdiction or improper venue. Pedestrian’s claim will be based on negligence, and therefore arises under state law.

The partner hands you the file in Pedestrian’s case. It yields the following information.

Pedestrian is a doctor who practices in Fraser. She owns a home in Fraser, where she was born and reared and plans to live out her days. She was injured in the accident, and if liability can be established, Pedestrian could realistically hope to recover about $250,000.

David Driver is a law student who lived at his parent’s home in Fraser through high school. He next lived on campus while attending college at Fraser State University, and then moved to a small apartment in the state of Coffman when he became a student at Coffman Law School. Driver’s parents support him while he attends law school, and he has frequently returned to his parent’s home in Fraser during vacation periods. Driver is a third-year law student. Between his second and third years of law school, Driver was a summer associate for a law firm in Coffman. During the summer, he moved from the apartment he had rented for two years into the much larger apartment he now occupies.

Two months ago, Driver accepted a full‑time job with a law firm in the state of Northrop. He obtained the job through interviews at Coffman Law School, and has not visited Northrop since accepting the job. He says he plans to remain in Coffman only until his graduation five months hence. Just recently, Driver inherited a $120,000 house in Northrop from a cousin. He has never occupied or even seen the house, and it is being rented to tenants on a lease that expires in six months. Driver told the law firm in Northrop that if he got an offer he would move to Northrop and live there permanently.

D Corporation is a shoe manufacturer incorporated in the state of Coffman. Its home office and manufacturing plant are located in the state of Northrop. Consequently, its corporate officers and all of its employees are located in Northrop, although a few of its sales representatives occasionally travel in neighboring states to take orders for shoes. D Corporation sells most of its shoes in Northrop, but it sells a substantial number of shoes on a regular basis to wholesalers in both Coffman and Fraser. D Corporation has not sold any shoes or conducted any other business activities, directly or indirectly, in Elliott. The truck involved in the accident was passing through Elliott while transporting a shoe display to an industrial fair in another state. With the exception of property specifically described above, neither defendant owns any attachable property in any states.

Your initial research shows that all states involved in this case have adopted the general American rules of personal jurisdiction described in I.C. supra.

2. Fact Summary

Fraser is

— Pedestrian’s home state

— the home state of Driver’s parents

— one of the states in which D Corporation sells shoes

Coffman is

— the state where Driver is now attending law school

— one of the states in which D Corporation sells shoes

— the state in which D Corporation is incorporated

Elliott is

— the site of the accident

Northrop is

— the state of D Corporation’s plant and home office

— the state of Driver’s future law firm employer

— the state where the house inherited by Driver is located

— one of the states in which D Corporation sells shoes

3. Questions for Computer Exercise

Please answer “yes,” “no,” or “maybe” to each of the following questions. Assume that the defendants will raise every objection to jurisdiction and venue available to them, and that none of the courts will dismiss on forum non conveniens.

The “maybe” answer should be used when you believe a categorical “yes” or “no” is inappropriate. When more facts are needed before a decision can be made, or if sufficient facts are known but both sides have substantial legal arguments, then answer “maybe.” You should not base a “maybe” answer on conceivable but frivolous legal arguments or on factual possibilities that are unusual and improbable.

 

Q–1. COFFMAN–FEDERAL COURT

Can Pedestrian bring suit against both defendants in federal court in Coffman?

Answer yes, no, or maybe.

[Note: Your answer to this question should take into account all aspects of jurisdiction. The following questions will probe each of these aspects.]

1a. Would the court have subject matter jurisdiction?

Answer yes, no, or maybe.

Briefly state reasons for your answer.

1b. Would the court have personal (including quasi in rem) jurisdiction?

Answer yes, no, or maybe.

Reasons?

1c. Would venue be proper?

Answer yes, no, or maybe.

Reasons?

 

Q–2. COFFMAN–STATE COURT

Can Pedestrian bring suit against both defendants in state court in Coffman?

[Note: Since at least one court in the state system will be a court of general subject matter jurisdiction, and venue will always be proper in at least one state court, you need consider only whether the state has personal (including quasi in rem) jurisdiction to answer the state court questions.]

Answer yes, no, or maybe.

Reasons?

 

Q–3. ELLIOTT–FEDERAL COURT

Can Pedestrian bring suit against both defendants in federal court in Elliott?

3a. Would the court have subject matter jurisdiction?

Answer yes, no, or maybe.

Reasons?

3b. Would the court have personal (including quasi in rem) jurisdiction?

Answer yes, no, or maybe.

Reasons?

3c. Would venue be proper?

Answer yes, no, or maybe.

Reasons?

 

Q–4. ELLIOTT–STATE COURT

Can Pedestrian bring suit against both defendants in state court in Elliott?

Answer yes, no, or maybe.

Reasons?

 

Q–5. NORTHROP–FEDERAL COURT

Can Pedestrian bring suit against both defendants in federal court in Northrop?

5a. Would the court have subject matter jurisdiction?

Answer yes, no, or maybe.

Reasons?

5b. Would the court have personal (including quasi in rem) jurisdiction?

Answer yes, no, or maybe.

Reasons?

5c. Would venue be proper?

Answer yes, no, or maybe.

Reasons?

 

Q–6. NORTHROP–STATE COURT

Can Pedestrian bring suit against both defendants in state court in Northrop?

Answer yes, no, or maybe.

Reasons?

 

Q–7. FRASER–FEDERAL COURT

Can Pedestrian bring suit against both defendants in federal court in Fraser?

7a. Would the court have subject matter jurisdiction?

Answer yes, no, or maybe.

Reasons?

7b. Would the court have personal (including quasi in rem) jurisdiction?

Answer yes, no, or maybe.

Reasons?

7c. Would venue be proper?

Answer yes, no, or maybe.

Reasons?

 

Q–8. FRASER–STATE COURT

Can Pedestrian bring suit against both defendants in state court in Fraser?

Answer yes, no, or maybe.

Reasons?

This concludes the questions. You are now ready to go to the computer to work through CALI CIV 03: Jurisdiction and Venue.

__________

B. CALI CIV 19: Jurisdiction Over the Person

In addition to computer-assisted exercise, CALI CIV 03, you may wish to gain additional understanding of jurisdiction by doing the CALI CIV 19 “Jurisdiction over the Person” by James M. Klebba.[ix]

This exercise is designed for a student who has already read most of the material on personal jurisdiction in a typical first year civil procedure course. The topics covered include the “minimum contacts” test as a measure of the due process clause of the 14th Amendment, the interpretation and application of typical long-arm statutes, the interplay of statutory interpretation with the constitutional requirements, the difference between “specific” and “general” jurisdiction, the extent to which a defendant may contractually waive jurisdiction protections, an exploration of the different ways in which the jurisdictional rules apply depending on whether the defendant is an individual or a corporation, and the continued viability of the concept of “transient” jurisdiction. The above topics are explored through a series of hypotheticals, beginning with an extended variation on the facts of International Shoe. The two predominant jurisdictional statutes used in the exercise are the Uniform Interstate and International Procedure Act and the Rhode Island (California) statute that extends jurisdiction to the limits of the Due Process Clause. At appropriate points in the exercise students are able to refer back to the introductory fact situations, the Uniform Act, a list of important citations and previous related questions. The exercise is divided into three parts, so that one part can be conveniently done at a sitting.


[i]See Jack H. Friedenthal, Mary Kay Kane & Arthur R. Miller, Civil Procedure § 3.21 (4th ed. 2005) (discussing the Supreme Court’s series of pre-judgment seizure cases). (Return to text)

[ii]Further discussion of federal questions is beyond the scope of this brief note. The topic is explored in depth in a course in federal jurisdiction. See generally Charles A. Wright & Mary Kay Kane, The Law of Federal Courts § 17 (6th ed. 2002). (Return to text)

[iii]Under this provision, a corporation is said to have dual citizenship: state(s) of incorporation and the one state “where it has its principal place of business.” Since the statute offers no definition, different lines of authority emerged in the federal courts to interpret “principal place of business.” One line looked to the location of the executive offices or “nerve center” of the corporation. Another line looked to the state in which the corporation carried on the bulk of its activity. Some decisions attempted to blend the two lines of authority into a “total activity” test. The Supreme Court settled this matter by deciding the principal place of business of a corporation is the nerve center, the place where its high officers direct and control its activities. The Hertz Corp. v. Friend, 559 U.S. ___, 130 S. Ct. 1181, ___ L.Ed.2d ___ (2010). (Return to text)

[iv]See generally Charles A. Wright & Mary Kay Kane, The Law of Federal Courts §§ 32-37 (6th ed. 2002). (Return to text)

[v]See generally 14B Charles A. Wright, Arthur R. Miller, Edward H. Cooper & Joan E. Steinman, Federal Practice and Procedure § 3722.3 (4th ed. 2009).

The separate and independent claim statute, § 1441(c), encompasses only federal question cases (§ 1331). Diversity cases, part removable and part not, must remain in state court. This treatment has changed several times over the years. Prior to the most recent amendment in 1990, § 1441(c) included both federal question and diversity cases. What constitutes a “separate and independent claim” was construed narrowly in American Fire & Cas. Co. v. Finn, 341 U.S. 5, 71 S. Ct. 534, 95 L.Ed. 702 (1951). Consequently, claims are rarely removable as “separate and independent.”

As it is written today, § 1441(c) may well be unconstitutional. Because a “claim” is broadly construed to include an entire Constitutional case, which is essentially a single set of facts, a separate claim must be factually unrelated. Such an unrelated state law claim, while calling § 1441(c) into operation, would be beyond the scope of the Constitutional case allowed into federal court under supplemental jurisdiction under 28 U.S.C. § 1367 and indeed would exceed the judicial power allowed by Article III, § 2 of the Constitution. (Return to text)

[vi]The categories of the Uniform Interstate and International Procedure Act can be seen as designed to confer personal jurisdiction over nonresidents in situations in which the state has a legitimate interest in adjudicating the action and is likely to be a fair and convenient location for the lawsuit. For example, consider § 1.03(a)(3), conferring jurisdiction over a person who causes tortious injury by an act or omission in the state. This provision would apply in a case in which a civil action is brought against a nonresident who had allegedly committed assault and battery within the state. The state has a legitimate interest in keeping peace within its boundaries. Moreover, it is likely to be a convenient place to try the action, since the witnesses to the incident and to any resultant injury are likely to be located within the state.

The justification for jurisdiction is weaker in the situation covered by § 1.03(a)(4), which deals with injury caused within the state by a tort committed outside the state. This provision would apply to a defendant that manufactured a product outside the state and shipped it into the state, where it caused injury. The evidence concerning alleged negligent manufacture of the product is likely to be located outside of the state. On the other hand, evidence relating to the plaintiff’s injury is likely to be located in the state; the state has a legitimate interest in insuring that products within its borders are safe; and the requirement that the defendant have engaged in a persistent course of conduct in the state means that the defendant has had contacts with the state and should have been able to foresee the possibility of being sued there. (Return to text)

[vii]See Douglas D. McFarland, Dictum Run Wild: How Long-arm Statutes Extended to the Limits of Due Process, 84 Boston U. L. Rev. 491 (2004). This article finds that today eighteen states have enumerated acts statutes that are interpreted by their limited language, twelve states have enumerated acts statutes that are interpreted to extend to the limits of due process, nine states have enumerated acts statutes that have an additional limits of due process clause, and eleven states have limits of due process statutes. (Return to text)

[viii]In Asahi Metal Industry Co. v. Superior Court, 480 U.S. 102, 107 S. Ct. 1026, 94 L.Ed.2d 92 (1987), the Court splintered on the stream of commerce theory. Four Justices concluded that the theory is not constitutionally acceptable because a defendant by merely putting a product into the stream of commerce does not purposefully direct its activities at a particular state. Four Justices disagreed and concluded that the stream of commerce theory is constitutionally valid, but concurred in dismissal because jurisdiction over the defendant did not meet fair play and substantial justice. The ninth Justice decided that jurisdiction was not fair, but expressed an opinion that selling a large number of units annually into a state would amount to a purposeful availment. Consequently, four Justices rejected the stream of commerce theory while five accepted it, although the Court was unanimous for dismissal. Since Asahi, all but one of the nine Justices have been replaced on the Court, so the constitutional status of the stream of commerce remains murky. (Return to text)

[ix]Victor H. Schiro Professor of Law, Loyola University School of Law, New Orleans. The exercise is used with permission of the author. (Return to text)

License

Icon for the Creative Commons Attribution-NonCommercial-ShareAlike 4.0 International License

Computer-Aided Exercises in Civil Procedure by Center for Computer-Assisted Legal Instruction is licensed under a Creative Commons Attribution-NonCommercial-ShareAlike 4.0 International License, except where otherwise noted.